How can I prove that $(a_1+a_2+dotsb+a_n)(frac{1}{a_1}+frac{1}{a_2}+dotsb+frac{1}{a_n})geq n^2$...

What is it called when someone votes for an option that's not their first choice?

Should a narrator ever describe things based on a character's view instead of facts?

Sort with assumptions

Why is participating in the European Parliamentary elections used as a threat?

Derivative of an interpolated function

How to split IPA spelling into syllables

What is the meaning of "You've never met a graph you didn't like?"

Do people actually use the word "kaputt" in conversation?

Why can't I get pgrep output right to variable on bash script?

Non-Borel set in arbitrary metric space

Output visual diagram of picture

How to test the sharpness of a knife?

Travelling in US for more than 90 days

How do you say "Trust your struggle." in French?

Magnifying glass in hyperbolic space

Why does a 97 / 92 key piano exist by Bosendorfer?

When is the exact date for EOL of Ubuntu 14.04 LTS?

What is the purpose of using a decision tree?

Offset in split text content

Why is implicit conversion not ambiguous for non-primitive types?

How can I, as DM, avoid the Conga Line of Death occurring when implementing some form of flanking rule?

Can you describe someone as luxurious? As in someone who likes luxurious things?

What can I do if I am asked to learn different programming languages very frequently?

Extract substring according to regexp with sed or grep



How can I prove that $(a_1+a_2+dotsb+a_n)(frac{1}{a_1}+frac{1}{a_2}+dotsb+frac{1}{a_n})geq n^2$ [duplicate]


Proof that $left(sum^n_{k=1}x_kright)left(sum^n_{k=1}y_kright)geq n^2$Request for historical precedent of identity implying AM-HM inequalityProve that $frac{a_1^2}{a_1+a_2}+frac{a_2^2}{a_2+a_3}+ cdots frac{a_n^2}{a_n+a_1} geq frac12$How to prove $a_1^m + a_2^m + cdots + a_n^m geq frac{1}{a_1} + frac{1}{a_2} + cdots + frac{1}{a_n}$Prove that: $sqrt[3]{a_1^3+ a_2^3 +cdots+a_n^3} le sqrt{a_1^2 + a_2^2 +cdots+a_n^2}$If $a_igeq 0,$ prove $sumlimits_{n=1}^inftyfrac{a_1+a_2+cdots+a_n}{n}$diverges.$a_1,a_2,…,a_n$ are positive real numbers, their product is equal to $1$, show: $sum_{i=1}^n a_i^{frac 1 i} geq frac{n+1}2$Prove inequality $frac{a_1a_2…a_n}{(a_1+a_2+…+a_n)^n}le frac{(1-a_1)(1-a_2)…(1-a_n)}{(n-a_1-a_2-…-a_n)^n}$Proving the inequality $frac{1}{1+a_1} + frac{2}{(1+a_1)(1+a_2)} + cdots + frac{n}{(1+a_1)ldots (1+a_n)} < 2$Prove by induction $frac{a_1+a_2+cdots+a_n}{n}geq 1$Inequality : $ (a_1a_2+a_2a_3+ldots+a_na_1)left(frac{a_1}{a^2_2+a_2}+frac{a_2}{a^2_3+a_3}+ ldots+frac{a_n}{a^2_1+a_1}right)geq frac{n}{n+1} $How to prove $(a_1+a_2+dots a_n)left(frac{1}{a_1}+frac{1}{a_2}+dots+frac{1}{a_n}right)ge n^2$?













3












$begingroup$



This question already has an answer here:




  • Proof that $left(sum^n_{k=1}x_kright)left(sum^n_{k=1}y_kright)geq n^2$

    7 answers




I've been struggling for several hours, trying to prove this horrible inequality:
$(a_1+a_2+dotsb+a_n)left(frac{1}{a_1}+frac{1}{a_2}+dotsb+frac{1}{a_n}right)geq n^2$.



Where each $a_i$'s are positive and $n$ is a natural number.



First I tried the usual "mathematical induction" method, but it made no avail, since I could not show it would be true if $n=k+1$.



Suppose the inequality holds true when $n=k$, i.e.,



$(a_1+a_2+dotsb+a_k)left(frac{1}{a_1}+frac{1}{a_2}+dotsb+frac{1}{a_k}right)geq n^2$.



This is true if and only if



$(a_1+a_2+dotsb+a_k+a_{k+1})left(frac{1}{a_1}+frac{1}{a_2}+dotsb+frac{1}{a_k}+frac{1}{a_{k+1}}right) -a_{k+1}left(frac{1}{a_1}+dotsb+frac{1}{a_k}right)-frac{1}{a_{k+1}}(a_1+dotsb+a_k)-frac{a_{k+1}}{a_{k+1}} geq n^2$.



And I got stuck here.



The question looks like I have to use AM-GM inequality at some point, but I do not have a clue. Any small hints and clues will be appreciated.










share|cite|improve this question











$endgroup$



marked as duplicate by Arnaud D., qwr, Martin R, Asaf Karagila Mar 12 at 20:18


This question has been asked before and already has an answer. If those answers do not fully address your question, please ask a new question.


















  • $begingroup$
    Conditions on $a_i$?
    $endgroup$
    – Parcly Taxel
    Mar 12 at 15:02










  • $begingroup$
    whoa, I forgot the most important info there. They are all positive, and n is a natural number.
    $endgroup$
    – Ko Byeongmin
    Mar 12 at 15:03






  • 4




    $begingroup$
    Try Cauchy-Schwarz inequality?
    $endgroup$
    – Sik Feng Cheong
    Mar 12 at 15:04










  • $begingroup$
    Now I get it, I learn something new every day!! Thanks a lot :D
    $endgroup$
    – Ko Byeongmin
    Mar 12 at 15:05
















3












$begingroup$



This question already has an answer here:




  • Proof that $left(sum^n_{k=1}x_kright)left(sum^n_{k=1}y_kright)geq n^2$

    7 answers




I've been struggling for several hours, trying to prove this horrible inequality:
$(a_1+a_2+dotsb+a_n)left(frac{1}{a_1}+frac{1}{a_2}+dotsb+frac{1}{a_n}right)geq n^2$.



Where each $a_i$'s are positive and $n$ is a natural number.



First I tried the usual "mathematical induction" method, but it made no avail, since I could not show it would be true if $n=k+1$.



Suppose the inequality holds true when $n=k$, i.e.,



$(a_1+a_2+dotsb+a_k)left(frac{1}{a_1}+frac{1}{a_2}+dotsb+frac{1}{a_k}right)geq n^2$.



This is true if and only if



$(a_1+a_2+dotsb+a_k+a_{k+1})left(frac{1}{a_1}+frac{1}{a_2}+dotsb+frac{1}{a_k}+frac{1}{a_{k+1}}right) -a_{k+1}left(frac{1}{a_1}+dotsb+frac{1}{a_k}right)-frac{1}{a_{k+1}}(a_1+dotsb+a_k)-frac{a_{k+1}}{a_{k+1}} geq n^2$.



And I got stuck here.



The question looks like I have to use AM-GM inequality at some point, but I do not have a clue. Any small hints and clues will be appreciated.










share|cite|improve this question











$endgroup$



marked as duplicate by Arnaud D., qwr, Martin R, Asaf Karagila Mar 12 at 20:18


This question has been asked before and already has an answer. If those answers do not fully address your question, please ask a new question.


















  • $begingroup$
    Conditions on $a_i$?
    $endgroup$
    – Parcly Taxel
    Mar 12 at 15:02










  • $begingroup$
    whoa, I forgot the most important info there. They are all positive, and n is a natural number.
    $endgroup$
    – Ko Byeongmin
    Mar 12 at 15:03






  • 4




    $begingroup$
    Try Cauchy-Schwarz inequality?
    $endgroup$
    – Sik Feng Cheong
    Mar 12 at 15:04










  • $begingroup$
    Now I get it, I learn something new every day!! Thanks a lot :D
    $endgroup$
    – Ko Byeongmin
    Mar 12 at 15:05














3












3








3


1



$begingroup$



This question already has an answer here:




  • Proof that $left(sum^n_{k=1}x_kright)left(sum^n_{k=1}y_kright)geq n^2$

    7 answers




I've been struggling for several hours, trying to prove this horrible inequality:
$(a_1+a_2+dotsb+a_n)left(frac{1}{a_1}+frac{1}{a_2}+dotsb+frac{1}{a_n}right)geq n^2$.



Where each $a_i$'s are positive and $n$ is a natural number.



First I tried the usual "mathematical induction" method, but it made no avail, since I could not show it would be true if $n=k+1$.



Suppose the inequality holds true when $n=k$, i.e.,



$(a_1+a_2+dotsb+a_k)left(frac{1}{a_1}+frac{1}{a_2}+dotsb+frac{1}{a_k}right)geq n^2$.



This is true if and only if



$(a_1+a_2+dotsb+a_k+a_{k+1})left(frac{1}{a_1}+frac{1}{a_2}+dotsb+frac{1}{a_k}+frac{1}{a_{k+1}}right) -a_{k+1}left(frac{1}{a_1}+dotsb+frac{1}{a_k}right)-frac{1}{a_{k+1}}(a_1+dotsb+a_k)-frac{a_{k+1}}{a_{k+1}} geq n^2$.



And I got stuck here.



The question looks like I have to use AM-GM inequality at some point, but I do not have a clue. Any small hints and clues will be appreciated.










share|cite|improve this question











$endgroup$





This question already has an answer here:




  • Proof that $left(sum^n_{k=1}x_kright)left(sum^n_{k=1}y_kright)geq n^2$

    7 answers




I've been struggling for several hours, trying to prove this horrible inequality:
$(a_1+a_2+dotsb+a_n)left(frac{1}{a_1}+frac{1}{a_2}+dotsb+frac{1}{a_n}right)geq n^2$.



Where each $a_i$'s are positive and $n$ is a natural number.



First I tried the usual "mathematical induction" method, but it made no avail, since I could not show it would be true if $n=k+1$.



Suppose the inequality holds true when $n=k$, i.e.,



$(a_1+a_2+dotsb+a_k)left(frac{1}{a_1}+frac{1}{a_2}+dotsb+frac{1}{a_k}right)geq n^2$.



This is true if and only if



$(a_1+a_2+dotsb+a_k+a_{k+1})left(frac{1}{a_1}+frac{1}{a_2}+dotsb+frac{1}{a_k}+frac{1}{a_{k+1}}right) -a_{k+1}left(frac{1}{a_1}+dotsb+frac{1}{a_k}right)-frac{1}{a_{k+1}}(a_1+dotsb+a_k)-frac{a_{k+1}}{a_{k+1}} geq n^2$.



And I got stuck here.



The question looks like I have to use AM-GM inequality at some point, but I do not have a clue. Any small hints and clues will be appreciated.





This question already has an answer here:




  • Proof that $left(sum^n_{k=1}x_kright)left(sum^n_{k=1}y_kright)geq n^2$

    7 answers








analysis inequality






share|cite|improve this question















share|cite|improve this question













share|cite|improve this question




share|cite|improve this question








edited Mar 12 at 20:15









Asaf Karagila

306k33438769




306k33438769










asked Mar 12 at 15:02









Ko ByeongminKo Byeongmin

1546




1546




marked as duplicate by Arnaud D., qwr, Martin R, Asaf Karagila Mar 12 at 20:18


This question has been asked before and already has an answer. If those answers do not fully address your question, please ask a new question.









marked as duplicate by Arnaud D., qwr, Martin R, Asaf Karagila Mar 12 at 20:18


This question has been asked before and already has an answer. If those answers do not fully address your question, please ask a new question.














  • $begingroup$
    Conditions on $a_i$?
    $endgroup$
    – Parcly Taxel
    Mar 12 at 15:02










  • $begingroup$
    whoa, I forgot the most important info there. They are all positive, and n is a natural number.
    $endgroup$
    – Ko Byeongmin
    Mar 12 at 15:03






  • 4




    $begingroup$
    Try Cauchy-Schwarz inequality?
    $endgroup$
    – Sik Feng Cheong
    Mar 12 at 15:04










  • $begingroup$
    Now I get it, I learn something new every day!! Thanks a lot :D
    $endgroup$
    – Ko Byeongmin
    Mar 12 at 15:05


















  • $begingroup$
    Conditions on $a_i$?
    $endgroup$
    – Parcly Taxel
    Mar 12 at 15:02










  • $begingroup$
    whoa, I forgot the most important info there. They are all positive, and n is a natural number.
    $endgroup$
    – Ko Byeongmin
    Mar 12 at 15:03






  • 4




    $begingroup$
    Try Cauchy-Schwarz inequality?
    $endgroup$
    – Sik Feng Cheong
    Mar 12 at 15:04










  • $begingroup$
    Now I get it, I learn something new every day!! Thanks a lot :D
    $endgroup$
    – Ko Byeongmin
    Mar 12 at 15:05
















$begingroup$
Conditions on $a_i$?
$endgroup$
– Parcly Taxel
Mar 12 at 15:02




$begingroup$
Conditions on $a_i$?
$endgroup$
– Parcly Taxel
Mar 12 at 15:02












$begingroup$
whoa, I forgot the most important info there. They are all positive, and n is a natural number.
$endgroup$
– Ko Byeongmin
Mar 12 at 15:03




$begingroup$
whoa, I forgot the most important info there. They are all positive, and n is a natural number.
$endgroup$
– Ko Byeongmin
Mar 12 at 15:03




4




4




$begingroup$
Try Cauchy-Schwarz inequality?
$endgroup$
– Sik Feng Cheong
Mar 12 at 15:04




$begingroup$
Try Cauchy-Schwarz inequality?
$endgroup$
– Sik Feng Cheong
Mar 12 at 15:04












$begingroup$
Now I get it, I learn something new every day!! Thanks a lot :D
$endgroup$
– Ko Byeongmin
Mar 12 at 15:05




$begingroup$
Now I get it, I learn something new every day!! Thanks a lot :D
$endgroup$
– Ko Byeongmin
Mar 12 at 15:05










3 Answers
3






active

oldest

votes


















12












$begingroup$

Hint: AM-GM implies
$$
a_1+a_2+cdots +a_nge nsqrt[n]{a_1a_2cdots a_n}
$$
and $$
frac1{a_1}+frac1{a_2}+cdots +frac1{a_n}ge frac{n}{sqrt[n]{a_1a_2cdots a_n}}.
$$






share|cite|improve this answer









$endgroup$









  • 1




    $begingroup$
    That's a really strong hint, it looks like an answer
    $endgroup$
    – enedil
    Mar 12 at 16:56










  • $begingroup$
    You may be right.. It makes the remaining step look so trivial, so it can be justly regarded as an answer.
    $endgroup$
    – Song
    Mar 12 at 17:09



















9












$begingroup$

It is AM-HM inequality
$$frac{a_1+a_2+a_3+...+a_n}{n}geq frac{n}{frac{1}{a_1}+frac{1}{a_2}+frac{1}{a_3}+...+frac{1}{a_n}}$$






share|cite|improve this answer









$endgroup$





















    2












    $begingroup$

    Here is the proof by induction
    that you wanted.



    I added a more exact
    version of
    the identity used
    in the proof
    at the end.



    Let
    $s_n
    =u_nv_n
    $

    where
    $u_n=sum_{k=1}^n a_k,
    v_n= sum_{k=1}^n dfrac1{a_k}
    $
    .
    Then,
    assuming
    $s_n ge n^2$,



    $begin{array}\
    s_{n+1}
    &=u_{n+1}v_{n+1}\
    &=(u_n+a_{n+1}) (v_n+dfrac1{a_{n+1}})\
    &=u_nv_n+u_ndfrac1{a_{n+1}}+a_{n+1}v_n+1\
    &=s_n+u_ndfrac1{a_{n+1}}+a_{n+1}v_n+1\
    &ge n^2+u_ndfrac1{a_{n+1}}+a_{n+1}v_n+1\
    end{array}
    $



    So it is sufficient
    to show that
    $u_ndfrac1{a_{n+1}}+v_na_{n+1}
    ge 2n
    $
    .



    By simple algebra,
    if $a, b ge 0$ then
    $a+b
    ge 2sqrt{ab}
    $
    .
    (Rewrite as
    $(sqrt{a}-sqrt{b})^2ge 0$
    or,
    as an identity,
    $a+b
    =2sqrt{ab}+(sqrt{a}-sqrt{b})^2$
    .)



    Therefore



    $begin{array}\
    u_ndfrac1{a_{n+1}}+v_na_{n+1}
    &ge sqrt{(u_ndfrac1{a_{n+1}})(v_na_{n+1})}\
    &= sqrt{u_nv_n}\
    &=2sqrt{s_n}\
    &ge 2sqrt{n^2}
    qquadtext{by the induction hypothesis}\
    &=2n\
    end{array}
    $



    and we are done.



    I find it interesting that
    $s_n ge n^2$
    is used twice in the
    induction step.



    Note that,
    if we use the identity above,
    $a+b
    =2sqrt{ab}+(sqrt{a}-sqrt{b})^2$
    ,
    we get this:



    $begin{array}\
    s_{n+1}
    &=s_n+u_ndfrac1{a_{n+1}}+a_{n+1}v_n+1\
    &=s_n+2sqrt{u_ndfrac1{a_{n+1}}a_{n+1}v_n}+1+(sqrt{u_ndfrac1{a_{n+1}}}-sqrt{a_{n+1}v_n})^2\
    &=s_n+2sqrt{s_n}+1+dfrac1{a_{n+1}}(sqrt{u_n}-a_{n+1}sqrt{v_n})^2\
    &=(sqrt{s_n}+1)^2+dfrac1{a_{n+1}}(sqrt{u_n}-a_{n+1}sqrt{v_n})^2\
    &ge(sqrt{s_n}+1)^2\
    end{array}
    $



    with equality
    if and only if
    $a_{n+1}
    =sqrt{dfrac{u_n}{v_n}}
    =sqrt{dfrac{sum_{k=1}^n a_k}{sum_{k=1}^n dfrac1{a_k}}}
    $
    .






    share|cite|improve this answer











    $endgroup$









    • 1




      $begingroup$
      This is wonderful! I love it that you’ve done it without using any “machineries” like the AM-GM inequality. I hope one day I become skillful as you are! Have a great day :)
      $endgroup$
      – Ko Byeongmin
      Mar 12 at 23:46










    • $begingroup$
      Thank you. This makes my day.
      $endgroup$
      – marty cohen
      Mar 13 at 1:10






    • 1




      $begingroup$
      Shouldn't $a+b =2ab+(sqrt{a}-sqrt{b})^2$ be $a+b =2sqrt{ab}+(sqrt{a}-sqrt{b})^2$?
      $endgroup$
      – Martin R
      Mar 16 at 7:54










    • $begingroup$
      Yes. Thank you. Corrected and comment upvoted.
      $endgroup$
      – marty cohen
      Mar 16 at 20:57


















    3 Answers
    3






    active

    oldest

    votes








    3 Answers
    3






    active

    oldest

    votes









    active

    oldest

    votes






    active

    oldest

    votes









    12












    $begingroup$

    Hint: AM-GM implies
    $$
    a_1+a_2+cdots +a_nge nsqrt[n]{a_1a_2cdots a_n}
    $$
    and $$
    frac1{a_1}+frac1{a_2}+cdots +frac1{a_n}ge frac{n}{sqrt[n]{a_1a_2cdots a_n}}.
    $$






    share|cite|improve this answer









    $endgroup$









    • 1




      $begingroup$
      That's a really strong hint, it looks like an answer
      $endgroup$
      – enedil
      Mar 12 at 16:56










    • $begingroup$
      You may be right.. It makes the remaining step look so trivial, so it can be justly regarded as an answer.
      $endgroup$
      – Song
      Mar 12 at 17:09
















    12












    $begingroup$

    Hint: AM-GM implies
    $$
    a_1+a_2+cdots +a_nge nsqrt[n]{a_1a_2cdots a_n}
    $$
    and $$
    frac1{a_1}+frac1{a_2}+cdots +frac1{a_n}ge frac{n}{sqrt[n]{a_1a_2cdots a_n}}.
    $$






    share|cite|improve this answer









    $endgroup$









    • 1




      $begingroup$
      That's a really strong hint, it looks like an answer
      $endgroup$
      – enedil
      Mar 12 at 16:56










    • $begingroup$
      You may be right.. It makes the remaining step look so trivial, so it can be justly regarded as an answer.
      $endgroup$
      – Song
      Mar 12 at 17:09














    12












    12








    12





    $begingroup$

    Hint: AM-GM implies
    $$
    a_1+a_2+cdots +a_nge nsqrt[n]{a_1a_2cdots a_n}
    $$
    and $$
    frac1{a_1}+frac1{a_2}+cdots +frac1{a_n}ge frac{n}{sqrt[n]{a_1a_2cdots a_n}}.
    $$






    share|cite|improve this answer









    $endgroup$



    Hint: AM-GM implies
    $$
    a_1+a_2+cdots +a_nge nsqrt[n]{a_1a_2cdots a_n}
    $$
    and $$
    frac1{a_1}+frac1{a_2}+cdots +frac1{a_n}ge frac{n}{sqrt[n]{a_1a_2cdots a_n}}.
    $$







    share|cite|improve this answer












    share|cite|improve this answer



    share|cite|improve this answer










    answered Mar 12 at 15:06









    SongSong

    18.3k21549




    18.3k21549








    • 1




      $begingroup$
      That's a really strong hint, it looks like an answer
      $endgroup$
      – enedil
      Mar 12 at 16:56










    • $begingroup$
      You may be right.. It makes the remaining step look so trivial, so it can be justly regarded as an answer.
      $endgroup$
      – Song
      Mar 12 at 17:09














    • 1




      $begingroup$
      That's a really strong hint, it looks like an answer
      $endgroup$
      – enedil
      Mar 12 at 16:56










    • $begingroup$
      You may be right.. It makes the remaining step look so trivial, so it can be justly regarded as an answer.
      $endgroup$
      – Song
      Mar 12 at 17:09








    1




    1




    $begingroup$
    That's a really strong hint, it looks like an answer
    $endgroup$
    – enedil
    Mar 12 at 16:56




    $begingroup$
    That's a really strong hint, it looks like an answer
    $endgroup$
    – enedil
    Mar 12 at 16:56












    $begingroup$
    You may be right.. It makes the remaining step look so trivial, so it can be justly regarded as an answer.
    $endgroup$
    – Song
    Mar 12 at 17:09




    $begingroup$
    You may be right.. It makes the remaining step look so trivial, so it can be justly regarded as an answer.
    $endgroup$
    – Song
    Mar 12 at 17:09











    9












    $begingroup$

    It is AM-HM inequality
    $$frac{a_1+a_2+a_3+...+a_n}{n}geq frac{n}{frac{1}{a_1}+frac{1}{a_2}+frac{1}{a_3}+...+frac{1}{a_n}}$$






    share|cite|improve this answer









    $endgroup$


















      9












      $begingroup$

      It is AM-HM inequality
      $$frac{a_1+a_2+a_3+...+a_n}{n}geq frac{n}{frac{1}{a_1}+frac{1}{a_2}+frac{1}{a_3}+...+frac{1}{a_n}}$$






      share|cite|improve this answer









      $endgroup$
















        9












        9








        9





        $begingroup$

        It is AM-HM inequality
        $$frac{a_1+a_2+a_3+...+a_n}{n}geq frac{n}{frac{1}{a_1}+frac{1}{a_2}+frac{1}{a_3}+...+frac{1}{a_n}}$$






        share|cite|improve this answer









        $endgroup$



        It is AM-HM inequality
        $$frac{a_1+a_2+a_3+...+a_n}{n}geq frac{n}{frac{1}{a_1}+frac{1}{a_2}+frac{1}{a_3}+...+frac{1}{a_n}}$$







        share|cite|improve this answer












        share|cite|improve this answer



        share|cite|improve this answer










        answered Mar 12 at 15:07









        Dr. Sonnhard GraubnerDr. Sonnhard Graubner

        77.8k42866




        77.8k42866























            2












            $begingroup$

            Here is the proof by induction
            that you wanted.



            I added a more exact
            version of
            the identity used
            in the proof
            at the end.



            Let
            $s_n
            =u_nv_n
            $

            where
            $u_n=sum_{k=1}^n a_k,
            v_n= sum_{k=1}^n dfrac1{a_k}
            $
            .
            Then,
            assuming
            $s_n ge n^2$,



            $begin{array}\
            s_{n+1}
            &=u_{n+1}v_{n+1}\
            &=(u_n+a_{n+1}) (v_n+dfrac1{a_{n+1}})\
            &=u_nv_n+u_ndfrac1{a_{n+1}}+a_{n+1}v_n+1\
            &=s_n+u_ndfrac1{a_{n+1}}+a_{n+1}v_n+1\
            &ge n^2+u_ndfrac1{a_{n+1}}+a_{n+1}v_n+1\
            end{array}
            $



            So it is sufficient
            to show that
            $u_ndfrac1{a_{n+1}}+v_na_{n+1}
            ge 2n
            $
            .



            By simple algebra,
            if $a, b ge 0$ then
            $a+b
            ge 2sqrt{ab}
            $
            .
            (Rewrite as
            $(sqrt{a}-sqrt{b})^2ge 0$
            or,
            as an identity,
            $a+b
            =2sqrt{ab}+(sqrt{a}-sqrt{b})^2$
            .)



            Therefore



            $begin{array}\
            u_ndfrac1{a_{n+1}}+v_na_{n+1}
            &ge sqrt{(u_ndfrac1{a_{n+1}})(v_na_{n+1})}\
            &= sqrt{u_nv_n}\
            &=2sqrt{s_n}\
            &ge 2sqrt{n^2}
            qquadtext{by the induction hypothesis}\
            &=2n\
            end{array}
            $



            and we are done.



            I find it interesting that
            $s_n ge n^2$
            is used twice in the
            induction step.



            Note that,
            if we use the identity above,
            $a+b
            =2sqrt{ab}+(sqrt{a}-sqrt{b})^2$
            ,
            we get this:



            $begin{array}\
            s_{n+1}
            &=s_n+u_ndfrac1{a_{n+1}}+a_{n+1}v_n+1\
            &=s_n+2sqrt{u_ndfrac1{a_{n+1}}a_{n+1}v_n}+1+(sqrt{u_ndfrac1{a_{n+1}}}-sqrt{a_{n+1}v_n})^2\
            &=s_n+2sqrt{s_n}+1+dfrac1{a_{n+1}}(sqrt{u_n}-a_{n+1}sqrt{v_n})^2\
            &=(sqrt{s_n}+1)^2+dfrac1{a_{n+1}}(sqrt{u_n}-a_{n+1}sqrt{v_n})^2\
            &ge(sqrt{s_n}+1)^2\
            end{array}
            $



            with equality
            if and only if
            $a_{n+1}
            =sqrt{dfrac{u_n}{v_n}}
            =sqrt{dfrac{sum_{k=1}^n a_k}{sum_{k=1}^n dfrac1{a_k}}}
            $
            .






            share|cite|improve this answer











            $endgroup$









            • 1




              $begingroup$
              This is wonderful! I love it that you’ve done it without using any “machineries” like the AM-GM inequality. I hope one day I become skillful as you are! Have a great day :)
              $endgroup$
              – Ko Byeongmin
              Mar 12 at 23:46










            • $begingroup$
              Thank you. This makes my day.
              $endgroup$
              – marty cohen
              Mar 13 at 1:10






            • 1




              $begingroup$
              Shouldn't $a+b =2ab+(sqrt{a}-sqrt{b})^2$ be $a+b =2sqrt{ab}+(sqrt{a}-sqrt{b})^2$?
              $endgroup$
              – Martin R
              Mar 16 at 7:54










            • $begingroup$
              Yes. Thank you. Corrected and comment upvoted.
              $endgroup$
              – marty cohen
              Mar 16 at 20:57
















            2












            $begingroup$

            Here is the proof by induction
            that you wanted.



            I added a more exact
            version of
            the identity used
            in the proof
            at the end.



            Let
            $s_n
            =u_nv_n
            $

            where
            $u_n=sum_{k=1}^n a_k,
            v_n= sum_{k=1}^n dfrac1{a_k}
            $
            .
            Then,
            assuming
            $s_n ge n^2$,



            $begin{array}\
            s_{n+1}
            &=u_{n+1}v_{n+1}\
            &=(u_n+a_{n+1}) (v_n+dfrac1{a_{n+1}})\
            &=u_nv_n+u_ndfrac1{a_{n+1}}+a_{n+1}v_n+1\
            &=s_n+u_ndfrac1{a_{n+1}}+a_{n+1}v_n+1\
            &ge n^2+u_ndfrac1{a_{n+1}}+a_{n+1}v_n+1\
            end{array}
            $



            So it is sufficient
            to show that
            $u_ndfrac1{a_{n+1}}+v_na_{n+1}
            ge 2n
            $
            .



            By simple algebra,
            if $a, b ge 0$ then
            $a+b
            ge 2sqrt{ab}
            $
            .
            (Rewrite as
            $(sqrt{a}-sqrt{b})^2ge 0$
            or,
            as an identity,
            $a+b
            =2sqrt{ab}+(sqrt{a}-sqrt{b})^2$
            .)



            Therefore



            $begin{array}\
            u_ndfrac1{a_{n+1}}+v_na_{n+1}
            &ge sqrt{(u_ndfrac1{a_{n+1}})(v_na_{n+1})}\
            &= sqrt{u_nv_n}\
            &=2sqrt{s_n}\
            &ge 2sqrt{n^2}
            qquadtext{by the induction hypothesis}\
            &=2n\
            end{array}
            $



            and we are done.



            I find it interesting that
            $s_n ge n^2$
            is used twice in the
            induction step.



            Note that,
            if we use the identity above,
            $a+b
            =2sqrt{ab}+(sqrt{a}-sqrt{b})^2$
            ,
            we get this:



            $begin{array}\
            s_{n+1}
            &=s_n+u_ndfrac1{a_{n+1}}+a_{n+1}v_n+1\
            &=s_n+2sqrt{u_ndfrac1{a_{n+1}}a_{n+1}v_n}+1+(sqrt{u_ndfrac1{a_{n+1}}}-sqrt{a_{n+1}v_n})^2\
            &=s_n+2sqrt{s_n}+1+dfrac1{a_{n+1}}(sqrt{u_n}-a_{n+1}sqrt{v_n})^2\
            &=(sqrt{s_n}+1)^2+dfrac1{a_{n+1}}(sqrt{u_n}-a_{n+1}sqrt{v_n})^2\
            &ge(sqrt{s_n}+1)^2\
            end{array}
            $



            with equality
            if and only if
            $a_{n+1}
            =sqrt{dfrac{u_n}{v_n}}
            =sqrt{dfrac{sum_{k=1}^n a_k}{sum_{k=1}^n dfrac1{a_k}}}
            $
            .






            share|cite|improve this answer











            $endgroup$









            • 1




              $begingroup$
              This is wonderful! I love it that you’ve done it without using any “machineries” like the AM-GM inequality. I hope one day I become skillful as you are! Have a great day :)
              $endgroup$
              – Ko Byeongmin
              Mar 12 at 23:46










            • $begingroup$
              Thank you. This makes my day.
              $endgroup$
              – marty cohen
              Mar 13 at 1:10






            • 1




              $begingroup$
              Shouldn't $a+b =2ab+(sqrt{a}-sqrt{b})^2$ be $a+b =2sqrt{ab}+(sqrt{a}-sqrt{b})^2$?
              $endgroup$
              – Martin R
              Mar 16 at 7:54










            • $begingroup$
              Yes. Thank you. Corrected and comment upvoted.
              $endgroup$
              – marty cohen
              Mar 16 at 20:57














            2












            2








            2





            $begingroup$

            Here is the proof by induction
            that you wanted.



            I added a more exact
            version of
            the identity used
            in the proof
            at the end.



            Let
            $s_n
            =u_nv_n
            $

            where
            $u_n=sum_{k=1}^n a_k,
            v_n= sum_{k=1}^n dfrac1{a_k}
            $
            .
            Then,
            assuming
            $s_n ge n^2$,



            $begin{array}\
            s_{n+1}
            &=u_{n+1}v_{n+1}\
            &=(u_n+a_{n+1}) (v_n+dfrac1{a_{n+1}})\
            &=u_nv_n+u_ndfrac1{a_{n+1}}+a_{n+1}v_n+1\
            &=s_n+u_ndfrac1{a_{n+1}}+a_{n+1}v_n+1\
            &ge n^2+u_ndfrac1{a_{n+1}}+a_{n+1}v_n+1\
            end{array}
            $



            So it is sufficient
            to show that
            $u_ndfrac1{a_{n+1}}+v_na_{n+1}
            ge 2n
            $
            .



            By simple algebra,
            if $a, b ge 0$ then
            $a+b
            ge 2sqrt{ab}
            $
            .
            (Rewrite as
            $(sqrt{a}-sqrt{b})^2ge 0$
            or,
            as an identity,
            $a+b
            =2sqrt{ab}+(sqrt{a}-sqrt{b})^2$
            .)



            Therefore



            $begin{array}\
            u_ndfrac1{a_{n+1}}+v_na_{n+1}
            &ge sqrt{(u_ndfrac1{a_{n+1}})(v_na_{n+1})}\
            &= sqrt{u_nv_n}\
            &=2sqrt{s_n}\
            &ge 2sqrt{n^2}
            qquadtext{by the induction hypothesis}\
            &=2n\
            end{array}
            $



            and we are done.



            I find it interesting that
            $s_n ge n^2$
            is used twice in the
            induction step.



            Note that,
            if we use the identity above,
            $a+b
            =2sqrt{ab}+(sqrt{a}-sqrt{b})^2$
            ,
            we get this:



            $begin{array}\
            s_{n+1}
            &=s_n+u_ndfrac1{a_{n+1}}+a_{n+1}v_n+1\
            &=s_n+2sqrt{u_ndfrac1{a_{n+1}}a_{n+1}v_n}+1+(sqrt{u_ndfrac1{a_{n+1}}}-sqrt{a_{n+1}v_n})^2\
            &=s_n+2sqrt{s_n}+1+dfrac1{a_{n+1}}(sqrt{u_n}-a_{n+1}sqrt{v_n})^2\
            &=(sqrt{s_n}+1)^2+dfrac1{a_{n+1}}(sqrt{u_n}-a_{n+1}sqrt{v_n})^2\
            &ge(sqrt{s_n}+1)^2\
            end{array}
            $



            with equality
            if and only if
            $a_{n+1}
            =sqrt{dfrac{u_n}{v_n}}
            =sqrt{dfrac{sum_{k=1}^n a_k}{sum_{k=1}^n dfrac1{a_k}}}
            $
            .






            share|cite|improve this answer











            $endgroup$



            Here is the proof by induction
            that you wanted.



            I added a more exact
            version of
            the identity used
            in the proof
            at the end.



            Let
            $s_n
            =u_nv_n
            $

            where
            $u_n=sum_{k=1}^n a_k,
            v_n= sum_{k=1}^n dfrac1{a_k}
            $
            .
            Then,
            assuming
            $s_n ge n^2$,



            $begin{array}\
            s_{n+1}
            &=u_{n+1}v_{n+1}\
            &=(u_n+a_{n+1}) (v_n+dfrac1{a_{n+1}})\
            &=u_nv_n+u_ndfrac1{a_{n+1}}+a_{n+1}v_n+1\
            &=s_n+u_ndfrac1{a_{n+1}}+a_{n+1}v_n+1\
            &ge n^2+u_ndfrac1{a_{n+1}}+a_{n+1}v_n+1\
            end{array}
            $



            So it is sufficient
            to show that
            $u_ndfrac1{a_{n+1}}+v_na_{n+1}
            ge 2n
            $
            .



            By simple algebra,
            if $a, b ge 0$ then
            $a+b
            ge 2sqrt{ab}
            $
            .
            (Rewrite as
            $(sqrt{a}-sqrt{b})^2ge 0$
            or,
            as an identity,
            $a+b
            =2sqrt{ab}+(sqrt{a}-sqrt{b})^2$
            .)



            Therefore



            $begin{array}\
            u_ndfrac1{a_{n+1}}+v_na_{n+1}
            &ge sqrt{(u_ndfrac1{a_{n+1}})(v_na_{n+1})}\
            &= sqrt{u_nv_n}\
            &=2sqrt{s_n}\
            &ge 2sqrt{n^2}
            qquadtext{by the induction hypothesis}\
            &=2n\
            end{array}
            $



            and we are done.



            I find it interesting that
            $s_n ge n^2$
            is used twice in the
            induction step.



            Note that,
            if we use the identity above,
            $a+b
            =2sqrt{ab}+(sqrt{a}-sqrt{b})^2$
            ,
            we get this:



            $begin{array}\
            s_{n+1}
            &=s_n+u_ndfrac1{a_{n+1}}+a_{n+1}v_n+1\
            &=s_n+2sqrt{u_ndfrac1{a_{n+1}}a_{n+1}v_n}+1+(sqrt{u_ndfrac1{a_{n+1}}}-sqrt{a_{n+1}v_n})^2\
            &=s_n+2sqrt{s_n}+1+dfrac1{a_{n+1}}(sqrt{u_n}-a_{n+1}sqrt{v_n})^2\
            &=(sqrt{s_n}+1)^2+dfrac1{a_{n+1}}(sqrt{u_n}-a_{n+1}sqrt{v_n})^2\
            &ge(sqrt{s_n}+1)^2\
            end{array}
            $



            with equality
            if and only if
            $a_{n+1}
            =sqrt{dfrac{u_n}{v_n}}
            =sqrt{dfrac{sum_{k=1}^n a_k}{sum_{k=1}^n dfrac1{a_k}}}
            $
            .







            share|cite|improve this answer














            share|cite|improve this answer



            share|cite|improve this answer








            edited Mar 16 at 20:57

























            answered Mar 12 at 20:01









            marty cohenmarty cohen

            74.4k549129




            74.4k549129








            • 1




              $begingroup$
              This is wonderful! I love it that you’ve done it without using any “machineries” like the AM-GM inequality. I hope one day I become skillful as you are! Have a great day :)
              $endgroup$
              – Ko Byeongmin
              Mar 12 at 23:46










            • $begingroup$
              Thank you. This makes my day.
              $endgroup$
              – marty cohen
              Mar 13 at 1:10






            • 1




              $begingroup$
              Shouldn't $a+b =2ab+(sqrt{a}-sqrt{b})^2$ be $a+b =2sqrt{ab}+(sqrt{a}-sqrt{b})^2$?
              $endgroup$
              – Martin R
              Mar 16 at 7:54










            • $begingroup$
              Yes. Thank you. Corrected and comment upvoted.
              $endgroup$
              – marty cohen
              Mar 16 at 20:57














            • 1




              $begingroup$
              This is wonderful! I love it that you’ve done it without using any “machineries” like the AM-GM inequality. I hope one day I become skillful as you are! Have a great day :)
              $endgroup$
              – Ko Byeongmin
              Mar 12 at 23:46










            • $begingroup$
              Thank you. This makes my day.
              $endgroup$
              – marty cohen
              Mar 13 at 1:10






            • 1




              $begingroup$
              Shouldn't $a+b =2ab+(sqrt{a}-sqrt{b})^2$ be $a+b =2sqrt{ab}+(sqrt{a}-sqrt{b})^2$?
              $endgroup$
              – Martin R
              Mar 16 at 7:54










            • $begingroup$
              Yes. Thank you. Corrected and comment upvoted.
              $endgroup$
              – marty cohen
              Mar 16 at 20:57








            1




            1




            $begingroup$
            This is wonderful! I love it that you’ve done it without using any “machineries” like the AM-GM inequality. I hope one day I become skillful as you are! Have a great day :)
            $endgroup$
            – Ko Byeongmin
            Mar 12 at 23:46




            $begingroup$
            This is wonderful! I love it that you’ve done it without using any “machineries” like the AM-GM inequality. I hope one day I become skillful as you are! Have a great day :)
            $endgroup$
            – Ko Byeongmin
            Mar 12 at 23:46












            $begingroup$
            Thank you. This makes my day.
            $endgroup$
            – marty cohen
            Mar 13 at 1:10




            $begingroup$
            Thank you. This makes my day.
            $endgroup$
            – marty cohen
            Mar 13 at 1:10




            1




            1




            $begingroup$
            Shouldn't $a+b =2ab+(sqrt{a}-sqrt{b})^2$ be $a+b =2sqrt{ab}+(sqrt{a}-sqrt{b})^2$?
            $endgroup$
            – Martin R
            Mar 16 at 7:54




            $begingroup$
            Shouldn't $a+b =2ab+(sqrt{a}-sqrt{b})^2$ be $a+b =2sqrt{ab}+(sqrt{a}-sqrt{b})^2$?
            $endgroup$
            – Martin R
            Mar 16 at 7:54












            $begingroup$
            Yes. Thank you. Corrected and comment upvoted.
            $endgroup$
            – marty cohen
            Mar 16 at 20:57




            $begingroup$
            Yes. Thank you. Corrected and comment upvoted.
            $endgroup$
            – marty cohen
            Mar 16 at 20:57



            Popular posts from this blog

            Magento 2 - Add success message with knockout Planned maintenance scheduled April 23, 2019 at 23:30 UTC (7:30pm US/Eastern) Announcing the arrival of Valued Associate #679: Cesar Manara Unicorn Meta Zoo #1: Why another podcast?Success / Error message on ajax request$.widget is not a function when loading a homepage after add custom jQuery on custom themeHow can bind jQuery to current document in Magento 2 When template load by ajaxRedirect page using plugin in Magento 2Magento 2 - Update quantity and totals of cart page without page reload?Magento 2: Quote data not loaded on knockout checkoutMagento 2 : I need to change add to cart success message after adding product into cart through pluginMagento 2.2.5 How to add additional products to cart from new checkout step?Magento 2 Add error/success message with knockoutCan't validate Post Code on checkout page

            Fil:Tokke komm.svg

            Where did Arya get these scars? Unicorn Meta Zoo #1: Why another podcast? Announcing the arrival of Valued Associate #679: Cesar Manara Favourite questions and answers from the 1st quarter of 2019Why did Arya refuse to end it?Has the pronunciation of Arya Stark's name changed?Has Arya forgiven people?Why did Arya Stark lose her vision?Why can Arya still use the faces?Has the Narrow Sea become narrower?Does Arya Stark know how to make poisons outside of the House of Black and White?Why did Nymeria leave Arya?Why did Arya not kill the Lannister soldiers she encountered in the Riverlands?What is the current canonical age of Sansa, Bran and Arya Stark?